Questions tagged [real-analysis]

Real-valued functions of real variable, analytic properties of functions and sequences, limits, continuity, smoothness of these.

Filter by
Sorted by
Tagged with
1 vote
2 answers
253 views

An integral inequality?

Let $v \in C^\infty(\mathbb R)$ such that $1 \ge v \ge 0$ and $\int_{\mathbb R} v \, dx = 1$. I want to show that if $$\int_{\mathbb R} v |v''|^2 \, dx < + \infty. \tag{$\star$}$$ then $$ \int_{\...
aaragon's user avatar
  • 73
2 votes
0 answers
102 views

Find a function $f\geq 0$ such that $e^{-t[(x-\partial_x)\partial_x]^2} f$ is not non-negative for some $t\geq 0$

Consider the square of the Ornstein-Uhlenbeck operator $$A=[(x-\partial_x)\partial_x]^2=(x-\partial_x)\partial_x (x-\partial_x)\partial_x.$$ We know that $[(x-\partial_x)\partial_x]^2$ cannot be a ...
matilda's user avatar
  • 90
3 votes
2 answers
209 views

Extremum placement for two-variable function

While teaching Calculus 2, one of my students asked me the following Given 3 points $x_1$, $x_2$, $x_3$. Whether there exists one function $z = f(x,y)$ which has exactly 2 extremum and 1 saddle point:...
IscoBerlin's user avatar
1 vote
1 answer
139 views

Unable to understand an application of Minkowski's inequality

Consider the following exerpt from the paper "Non-linear Quantum Processes" by Segal: with the norm $\|F\|=\left(\int\|F(x)\|^p \, d x\right)^{1 / p}$, then the operator $T_1^{\prime}: F \...
matilda's user avatar
  • 90
-1 votes
1 answer
68 views

Regions when a concave function is smaller than another concave function

Let $f_1,f_2:[0,1]\mapsto\mathbb{R}$ be two bounded and concave functions. Assume $f_1(0)<f_2(0)$ and $f_1(1)<f_2(1)$. I want to investigate the set $\mathcal{X}\triangleq\{x\in[0,1]: f_1(x)>...
Eggplant's user avatar
5 votes
1 answer
214 views

A limit related to quasi-periodic function

Let us consider $V(x) = 2-\sin(x) - \sin(\sqrt{2} x)$ on $x\in \mathbb{R}$ so that $V(x)>0$ everywhere. One can see that $$ \frac{C_1}{t^2} \leq \min_{|x|\leq t} V(x)\leq \frac{C_2}{t^2} $$ ...
Sean's user avatar
  • 313
3 votes
0 answers
125 views

On the continuity with respect to the increasing convex order

For $p\ge 1$, let $\mathcal P_p(\mathbb R)$ be the set of probability measures on $\mathbb R$ of finite $p^{\rm th}$ moment. Denote by $W_p$ the Wasserstein metric of order $p$ and by $\preceq$ the ...
Fawen90's user avatar
  • 975
2 votes
0 answers
938 views

On a deceptively tricky calculus problem

Motivation for this question: If the operators $B_i'$ satisfy an inequality, prove that $B_1'+\dots B_n'$ also satisfies the same inequality Let $A$ be a non-constant operator acting on $C^...
matilda's user avatar
  • 90
3 votes
1 answer
286 views

Does this condition characterise intervals, among subsets of the real line?

For a real number, $c\in \left]0,1\right[$, consider the following property $\mathbf(\mathbf P_c\mathbf)$ of subsets $A$ of $\mathbb R$: $\mathbf(\mathbf P_c\mathbf)$ For every bounded set $B\subset \...
Pietro Majer's user avatar
  • 56.6k
0 votes
1 answer
41 views

Box dimension and graph of Hölder function

In Kamont "ON THE FRACTIONAL ANISOTROPIC WIENER FIELD" (found here : https://www.math.uni.wroc.pl/~pms/files/16.1/Article/16.1.6.pdf), on page 96, it is claimed that, if a function $f:I^{d}\...
BabaUtah's user avatar
11 votes
3 answers
1k views

Every function on reals a sum of two surjective real functions?

From this question, and the answer thereof, we can see that every real valued function on reals is a sum of two injective functions. Is the same true if we replace injectivity by surjectivity. For ...
vidyarthi's user avatar
  • 2,027
8 votes
1 answer
343 views

Special Schwartz function on the positive interval

Is there a Schwartz function $\zeta(t)$, defined on $\mathbb{R}$, satisfying the following: $\int \zeta(t)\: dt=1$, $\int t^k \zeta(t)\: dt=0$ for all $k\geq 1$, $\operatorname{supp}(\zeta)\subset (0,...
SnowRabbit's user avatar
2 votes
2 answers
255 views

Erdős–Sierpiński duality in locally compact Polish groups (e.g. $\mathbb{R}^n$)

Erdős–Sierpiński mapping for a locally compact Polish group $G$ is a bijection $f$ from $G$ to $G$ such that $A$ is a null set in $G$ with respect to the Haar measure if and only if $f(A)$ is a meager ...
Nugi's user avatar
  • 131
4 votes
3 answers
839 views

Can these identities for the Euler-Mascheroni constant be proven?

I stumbled upon these 4 limit/integral identities involving Euler's constant aka gamma (~0.5772). They appear to be valid based on inspection but I have no idea how to prove them. In addition, I have ...
Mitch's user avatar
  • 195
7 votes
2 answers
590 views

Countably representing all closed sets of positive measure

This may be a naive question, but I don't see an immediate argument. Question: Does there exist a sequence $\{C_m\}_{m=1}^\infty$ of Borel subsets of $[0,1]$ with positive Lebesgue measure $|C_m|>0$...
Bedovlat's user avatar
  • 1,939
2 votes
1 answer
123 views

How to calculate this integral of squared Tricomi hypergeometric function

How to solve this integral $$ \int_{0}^{\infty}r^2 e^{-\omega r^2}U(-\nu,\frac{3}{2},\omega r^2)^2 \mathrm{d}r $$ where $\omega>0$ and $\nu \in \mathbb{R} \setminus \left \{ \frac{n-1}{2}\mid n \in ...
tsukatsuki_sorano's user avatar
2 votes
1 answer
132 views

Boundedness of an exit time from a campact set

Let $n\geq 1$ and $v\in\mathcal{C}^1(\mathbb{R}^n,\mathbb{R}^n)$. For $x_0\in\mathcal{O}$, let $\big(x(t)\big)_{t\geq 0}$ be the solution of \begin{align*} & x(0)=x_0 \\ & \dot{x}=v(x). \end{...
G. Panel's user avatar
  • 557
8 votes
1 answer
563 views

What is the minimum of this functional?

Recently I encountered an inequality from mathematical analysis. Let $f(x)$ be twice continuously differentiable in $[0,1]$ with $f(0)=f(1)=0$, then for all $x\in(0,1),f(x)\neq 0$, show that:$$\int_{0}...
jdhejw's user avatar
  • 317
3 votes
2 answers
477 views

A problem about how dominated convergence is used in the analysis of variation

I'm reading Existence of solutions to a higher dimensional mean-field equation on manifolds and get stuck on Lemma6. When $\lambda>\Lambda_1$, with $\Lambda_1=(2 m-1) ! \operatorname{vol}\left(S^{2 ...
Elio Li's user avatar
  • 729
2 votes
2 answers
247 views

If $\inf\{b\in\mathbb{R}\mid\sum_{n=1}^{\infty}e^{-ax_n-by_n}<+\infty\}=1-a$ for all $a\in [0,1]$, does this equality hold for all $a\in\mathbb{R}$?

Let $\left\{x_n\right\}_{n=1}^{+\infty},\left\{y_n\right\}_{n=1}^{+\infty}\subset [0,+\infty)$ be two sequences of non-negative real numbers. Suppose there exist $\lambda\ge 1, c\ge 0$ such that $\...
YC Su's user avatar
  • 23
2 votes
2 answers
137 views

Upper bound estimation for second-order variable-coefficient ODE

I'm tackling a second-order linear ordinary differential equation with variable coefficients $a(t)$ and seek advice on estimating the upper bound of $y(t)$ s.t $|y(t)|\le M$. The equation in question ...
lming2's user avatar
  • 45
3 votes
2 answers
233 views

Can every $L^p$ function be written as the weak derivative of a Sobolev function?

Let $\mathbb B^n$ be the open unit ball in $\mathbb R^n$, and $g: \mathbb B^n \to \mathbb R^n$ a measurable function with $|g| \in L^p (\mathbb B^n)$. Does there exist some function $f$ in the Sobolev ...
Nate River's user avatar
  • 4,832
0 votes
1 answer
97 views

The sequence has a stationary accumulation point

Let $f:\mathbb{R}^n\rightarrow \mathbb{R}$ be a smooth (continuously differentiable), convex function with a non-empty set of minimizers and $\{x^k\}$ be a sequence such that (a) $\{x^k\}$ has an ...
Dat Ba Tran's user avatar
4 votes
1 answer
173 views

What are the possible blow up limits of an $L^1$ function?

Let $f: [0, 1] \to \mathbb R$ be an $L^1$ function. Define for each $r > 0$, the blow up $f_r:[0, 1] \to \mathbb R$ by $$f_r (x) := \frac{f(rx)}{r}.$$ Suppose $f_r$ converges in $L^1$ to some ...
Nate River's user avatar
  • 4,832
27 votes
1 answer
2k views

Is every real number in [0,1] a product of three (or more) Cantor set's numbers?

It is well known that every number $x$ in the unit interval $[0,1]$ is the arithmetic mean of two elements of the (triadic) Cantor set $C$. The way to see it I like the most: the Cantor set is the ...
Pietro Majer's user avatar
  • 56.6k
0 votes
0 answers
53 views

Implicit function theorem for non $C^1$ mappings

I know that the inverse function theorem can be proved for differentiable mappings (not $C^1$) by requiring that $Df(x)$ has everywhere maximum rank (here is the reference https://terrytao.wordpress....
Lorenzo Vanni's user avatar
8 votes
4 answers
620 views

Distributional derivatives are locally integrable implies the distribution is also locally integrable?

Let $T$ be a distribution on $\mathbb{R}^n$ such that there are functions $f_1,\ldots,f_n \in L^1_\text{loc}(\mathbb{R}^n)$ so that $\dfrac{\partial T}{\partial x_j} = f_j, \forall j=1,\ldots,n. $ My ...
Jinie's user avatar
  • 83
0 votes
0 answers
176 views

How to prove that the uniform limit of $C^k$ functions is $C^{k-1,1}$?

Already asked in SE but no response, I think it also reasonably belongs here. https://math.stackexchange.com/questions/4829428/uniform-convergence-of-ck-functions Basically what the title says, plus ...
Clara Torres-Latorre's user avatar
2 votes
1 answer
280 views

Are these conditions regarding products of consecutive terms in a sequence of positive numbers equivalent?

Assume $w_n$ is a bounded (weight) sequence of positive numbers. We want to consider products of consecutive terms in this sequence. For $i,j\in \mathbb{N}$, define $M_i^j = w_i w_{i+1}\cdots w_{i+j-1}...
David Walmsley's user avatar
18 votes
1 answer
1k views

Does summing divergent series using cutoff functions give consistent results?

One way to try to give a value $S$ to a divergent series $\sum_{n=1}^\infty a_n$ is with a smooth cutoff function: $$ S = \lim_{N\to\infty}\sum_{n=1}^\infty a_n \eta\left(\frac{n}{N}\right) $$ where $\...
not all wrong's user avatar
0 votes
1 answer
103 views

Explanation for Tauberian theorems for Laplace transform

I am struggling with the following theorem in Feller's book "Probability Theory and its Applications". The tauberian theorem is written as follow : Let $F : [0,\infty) \to \mathbb{R}$ of ...
NancyBoy's user avatar
  • 175
1 vote
1 answer
174 views

The function $G(x) =(4\pi t)^{-d/2} \int_{\mathbb{R}^d} e^{\frac{-|x-y|^2}{4t}}|y|^k dy$ can be controlled when $|x|\rightarrow \infty$

In this paper, Lemma 6, Pinsky proves that $$H(x) =(4\pi t)^{-d/2} \int_{\mathbb{R}^d} e^{\frac{-|x-y|^2}{4t}}(1+|y|)^m \, dy$$ attains its maximum in $x=0$ for $m<0$. This can also be proven using ...
Ilovemath's user avatar
  • 585
-1 votes
1 answer
109 views

Proving that $\max_{w \in B(z)} e^{f(w)} \leq Ce^{f(z)}$

Let $f : \mathbb R^2 \to \mathbb R $ be a smooth function statisfying $$ 0 < \alpha \leq \Delta f(w) \leq \beta < \infty, \ \ \forall w \in \mathbb R^2 $$ where $\Delta$ denotes the Laplace ...
J. Swail's user avatar
  • 347
2 votes
2 answers
290 views

$L^1$ norm for a product of cosines

Let $k$ be an integer and consider the function $$ f(t)=\prod_{i=1}^{k} \cos(3^{i-1}\pi t). $$ I'm interested in finding bounds for $\int_{0}^{1}|f(t)|dt$ in terms of $k$. The first idea that comes to ...
Itachi's user avatar
  • 167
3 votes
2 answers
601 views

Are there any functions $f$ beyond trivial examples where $\int f(x +f(x + f(x +\dotsb)))\,dx$ $= F(x+F(x+F(x +\dotsb))) + C$ for some function $F$?

Basically the title explains most of my question here. Purely out of curiosity (I have no real application), I am wondering if there are any "interesting" functions $f$ where we know of a ...
gdoug's user avatar
  • 149
-1 votes
2 answers
81 views

Limits of integral series

Suppose we have the series of functions: \begin{equation} F(x)=\sum_{n=1}^{\infty} f_n(x) \end{equation} where convergence is uniform. Additionally, consider the partial functions of the series: \...
george andrade's user avatar
0 votes
1 answer
158 views

A self-consistent equation that turns into a differential equation

Suppose the function $f(x,y)$ is defined on a small neighbourhood of $(0,0)$ in $\mathbb{R} \times [0,\infty)$ and satisfies the self consistent equation \begin{align*} & f(x,y) = \frac{1}{1-y} + ...
Tardis's user avatar
  • 1,087
4 votes
2 answers
1k views

Does a function exist which is not Riemann integrable and satisfies the given condition:

I am looking for a function $f:[0,1]\rightarrow \mathbb{R}$ which is not Riemann integrable such that $$\sum_{k=0}^n |f(x_k)-f(x_{k-1})|^2 <1$$ for every choice of $0=x_0\le x_1 \le \cdots \le x_n =...
Lakshmi Priya's user avatar
5 votes
2 answers
355 views

$C^1$ harmonic functions on a dense open set are globally harmonic

In a paper I am studying, at a certain point the authors introduce a function $u\in C^1(B_1,\mathbb{R})$ which is harmonic in a dense open subset $U$ of $B_1$. From this, they seem to conclude that $u$...
No-one's user avatar
  • 1,037
4 votes
2 answers
270 views

Implicit function theorem without uniqueness?

Imagine you are given $f(x,y) := y^2-\sin(x)^2$ and you want to answer the question, if there is a neighbourhood of $x=0$ such that $f(x,y(x))=0$ with $y(0)=0$. One idea that comes to mind is the ...
Daisy_Duck's user avatar
10 votes
5 answers
2k views

Extracting a common convergent indexing from an uncountable family of sequences

Let $\mathcal{A}$ be some uncountable index set and $X$ be some separable reflexive Banach space. For each $\alpha \in \mathcal{A}$, let \begin{equation} \{ x_n^{\alpha} \}_{n=1}^\infty \end{equation} ...
Isaac's user avatar
  • 2,749
4 votes
1 answer
163 views

Compact-open Topology for Partial Maps?

I asked the same question on MathStackExchange a month ago and received no answer. I feel that this would be more suitable for MathOverflow. Compact open topology is one of the most common ways of ...
Bumblebee's user avatar
  • 1,019
10 votes
1 answer
639 views

Can a nowhere locally Hölder function be differentiable almost everywhere?

Fix $0 < \alpha < 1$. Suppose $f$ is nowhere locally $\alpha$-Hölder continuous - that is, it is not $\alpha$-Hölder on any open subinterval of $\mathbb R$. Is it possible for $f$ to be ...
Nate River's user avatar
  • 4,832
2 votes
2 answers
255 views

Making sense of the limit $\lim\limits_{x \to y} T(x,y) $ for a tempered distribution $T$ on $\mathbb{R}^{2n}$

I already posted a similar question on MO and looked into the references therein. However, I cannot find a satisfactory answer for my question..So I ask here again in a more refined form. Let $T \in \...
Isaac's user avatar
  • 2,749
7 votes
1 answer
259 views

High dimensional Fekete's subadditive lemma: does $|\vec x_{n+m}|\leq |\vec x_n+\vec x_m|$ imply the convergence of $\{\vec x_n/n\}$?

Let $d\geq 1$ be a positive integer. If $\{\vec x_n\}_{n=1}^\infty$ is a sequence of $d$-dimensional vectors satisfying $$\lvert\vec x_{n+m}\rvert\leq \lvert\vec x_n+\vec x_m\rvert\qquad \text{for all ...
Feng's user avatar
  • 517
3 votes
1 answer
239 views

Is this constraint convex?

I have an optimization problem where the following constraint causes DCP Rule Error. $$e^{x_n} \leq B \log _2\left(1+\frac{e^{\rho_n} g_n^2}{\sum_{i=1}^{n-1} e^{\...
Mojtaba's user avatar
  • 31
5 votes
1 answer
211 views

Can a solution to this parameterized ODE converge to zero?

Does there exists some $\gamma \ge 0$ such that the solution to the following ODE converges to 0 as $t \to \infty$? $$y'(t) = \alpha y(t) - \gamma \sigma(t) (1-y^2(t))$$ We are also given y(0) = 2/3, $...
icecuber's user avatar
  • 330
0 votes
1 answer
70 views

Continuous selectors of a continuous multifunctin on a compact metric space

I am currently working on a continuous selector problem of multifunctions. I am trying to figure out if a continuous multifunction defined on a compact metric space always admit a continuous selector. ...
Saito's user avatar
  • 49
0 votes
0 answers
174 views

Mean Cauchy sequences

Let $X$ be a $\sigma$-finite measure space. A sequence of functions $f_n \in L^1 (X)$ is said to be Cauchy in mean if $$\lim_{K \to \infty} \limsup_{N, M \to \infty} \frac{1}{NM} \sum_{i = K+1}^{K + N}...
Nate River's user avatar
  • 4,832
3 votes
1 answer
183 views

If $f$ is a derivative and $f=g$ a.e. for some Riemman integrable function $g$, then can we obtain the Riemann integrability of $f$?

Let $a,b\in\mathbb R$ with $a<b$ and $f:[a,b]\to\mathbb R$. Assume that there exists a Riemann integrable function $g:[a,b]\to\mathbb R$ such that $f=g$ almost everywhere. Then we can NOT conclude ...
Fergns Qian's user avatar